Skip to Main Content

June 2007, Logical Reasoning 1, Question 9

Transcript

So we start out with the question stem, which the following, if true, would most seriously weaken the argument? The word weaken, no surprise, indicates a weakened question. So in order to weaken an argument you need to understand the argument's conclusion and evidence. So here, the conclusion of the argument comes at the end of the first sentence.

We can expect a reversal of this trend in the very near future. Now, in order to know what trend they're talking about, we have to go back to the first part of the first sentence. Although video game sales have increased steadily over the past three years, so that's the trend. They're saying that we're not going to continue to see a steady increase in video game sales.

Now the evidence follows. So historically, over three quarters of video games sold have been purchased by people in the 13 to 16 year old age group, and the number of people in this age group is expected to decline steadily over the next 10 years. The conclusion is that video game sales are gonna go down soon, or they're not gonna continue to rise anyway.

And the evidence for that is that most of the people who used to buy them in the past were in a particular age group, and that generation's not gonna have as many people in it as it used to. Now once you have the argument down in a weaken question, you wanna think about the argument's assumptions. The things that need to be true for the argument to be right but that haven't been said explicitly.

Now the main assumption here is one that you gonna see in a lot of questions on the LSAT. Basically, anytime you make a conclusion about what's gonna happen in the future on the basis of information about the past, well, it requires believing that the future is going to resemble the past, that things that were true in the past aren't gonna change.

Here, the main thing we need to not change is the people who purchase video games. We need it to continue being mostly 13 to 16 year olds. So if suddenly half of all games get purchased by grandmothers in their 80s, well then this argument isn't gonna work. Maybe they're going to make up for the fact that their grandchildren aren't buying video games.

Let's go to the answer choices looking for an answer that attacks that assumption. So answer choice A says most people 17 years or older have never purchased a video game. That actually helps the assumption out. It suggests that, yeah, once you turn 16 you don't tend to buy video games anymore. It's not gonna help us weaken the argument.

The answer isn't A. Answer B, video game rentals have declined over the past 3 years. So this answer choice doesn't really go anywhere. We would need to know why video game rentals are declining to know if this was going to affect our assumption. Rentals going down is a change, but is it a change that would affect the people buying video games?

If it turns out that the reason that the rentals are declining is because people are just not interested in video games anymore, that would help the argument out. If it turns out the reason that rentals are declining is that people decided to stop renting and they want to buy now, well, that would hurt the argument. Since we don't know why the video game rentals are going down answer choice B can't affect our assumption.

So we go to answer choice C. New technology will undoubtedly make entirely new entertainment options available over the next 10 years. At best, this is going to strengthen the argument too, because new entertainment options are probably gonna reduce the amount of video game sold, competition. We're trying to weaken the argument.

In other words, we're looking for something that's going to not reduce the number of video games sold. Answer choice D says that the number of different types of video games is unlikely to decrease in the near future. This is not gonna hurt the argument, because it's saying that things are unlikely to decrease, so they're probably not gonna change.

Our assumption is that things are not gonna change, so this helps the argument. That means that the only answer choice left must be our answer, answer choice E. On the day of the test, you wouldn't really need to confirm why it was the answer because, it's the only one left. But let's take a look. It says most of the people who've purchased video games over the past three years are over the age of 16.

So notice the original argument said that, historically, over three quarters of video games sold had been purchased by people 13 to 16. Historically is all the way back to whenever video games were invented, whenever the first caveman invented a video game. Or I don't know, maybe it was Newton who did that, but whoever it was who invented the video game.

So all the way up until now, historically, it's been 13 to 16 year olds buying them. But if over the last three years during that time when video games sales have gone up, actually, it's been people who are older who are buying them. Well, that suggests that the past isn't going to resemble the future. In the future, that proportion of over three quarters is going to keep shrinking. So we're gonna have some people buying video games who aren't in that age group.

So that hurts the assumption and thus it weakens the argument. E's our answer, move on to the next question.

Read full transcript